The Student Room Group

2012 Higher Physics Discussion

Scroll to see replies

Reply 700
Original post by tomctutor
Thankyou Webbster91 ive checked through your answers and they seem to agree with my attached worked solutions to within 1sf more or less!

Ive also included scans of the original paper
Please quote link to this thread as: http://www.thestudentroom.co.uk/showpost.php?p=37811443&postcount=694


Thnak you so much for posting the solutions! Think I may have got 96%!!:biggrin:
if tomctutor's answers are correct, i've got 94% :smile: so chuffed!
See for 22(a)(ii) as well, is the acceleration value negative or positive ?, I used a positive value but think that might be regarded as wrong physics.

Reckon though I'll get an A if the SQA lower the pass mark ever so slightly (literally like about 3 or 4 marks!), which they might do considering the horrible multi-choice.
Original post by tomctutor
I dont agree they will carry forward your obviously wrong answer and award you some points, as its an arithmetic error and not a physics clanger!


Is it? My teacher (and I'm not trying to prove you wrong here, I am just confused) said that as soon as you state wrong physics (e.g V = T/S) then the markers stop because you have stated Wrong Physics and hence stop marking there.
Original post by tomctutor
Thankyou Webbster91 ive checked through your answers and they seem to agree with my attached worked solutions to within 1sf more or less!

Ive also included scans of the original paper
Please quote link to this thread as: http://www.thestudentroom.co.uk/showpost.php?p=37811443&postcount=694


bloody useful thanks, my friends and I were discussing the first question, and I got 15.5km whereas they had 16 and 16.8 so nice to know someone else got the same....
Reply 705
Original post by S119234
See for 22(a)(ii) as well, is the acceleration value negative or positive ?, I used a positive value but think that might be regarded as wrong physics.

Reckon though I'll get an A if the SQA lower the pass mark ever so slightly (literally like about 3 or 4 marks!), which they might do considering the horrible multi-choice.


Hoping as well, didn't you also receive a 1 at sg?


This was posted from The Student Room's iPhone/iPad App
a1ebc72e.png

Not sure if this would count.
Reply 707
I have about 71-72% after my stupid mc mistakes :frown:

Anybody know how hard it is nowadays in order to have an successful appeal?


This was posted from The Student Room's iPhone/iPad App
Original post by Direct15
Hoping as well, didn't you also receive a 1 at sg?


Did indeed!, would love to finally get an A at Higher, here's hoping! :smile:

I'm in a similar situation to you right now, went through it and got about 72-73% on it, but considering I'm marking quite harshly, I think it might go up with odd half-marks for formula on questions I got wrong, could be the case with you, so not all hope is lost!
(edited 11 years ago)
Are the solutions that were posted all right? Because if so I got 93% and I'm pretty chuffed with that
Original post by Webbster91
Hey here is my solutions if any use, not sure if they are all correct so give me a shout if not. Hope it went well for everybody today.



1.E
2.A
3.C
4.C
5.C
6.D
7.B
8.D
9.B
10.C
11.B
12.E
13.D
14.E
15.D
16.A
17.A
18.D
19.B
20.B

21.a)i)15.6km 64degrees south of east
a)ii) v=s/t
= 15.6/1.25
=12.5km/hr
b)i) 15.6km at 64degrees south of east

b)ii) t=d/s
=33/22
=1.5 hours
v=s/t
=15.6/1.5
=10.4km/hr

22a)i) D=st
=3.06x20
=61.2m
ii)s=ut+1/2at^2
=15x1.53+1/2x(-9.8)x1.53^2
=22.95-11.47
=11.5m
b) The ball is more likely to hit the tree as the maximum height will occur sooner so less of a chance for the ball to clear the tree.

23.a)i) W=Qv
=1.6x10^-19x1.22x10^3
=1.95^-16J

ii) Ek=W
1/2mv^2=QV
V^2=1.95^-16/1.09x10^-25
V^2=1.80x109
V=42296m/s

b) Ft=mv
v=ft/m
v=0.07x60/750
change in v=5.6x10^-3m/s

c)Xenon as f=ma so if acceleration is kept constant then the force exerted will be larger as mass is larger

24.a)k=p/t
=0.347 roughly for most so as there is a constant then temperature is directly proprtional to pressure

b) As the temperature is increased the particles have greater kinetic energy. This increase the speed of the particles and cause more frequent and more forceful collisions with the walls of the container. As the force exerted on the walls increase and the volume remains constant then the pressure increases.

c) So that all the gas inside the beaker has the same temperature.

25.a)i) I=v/r
=12/8
=1.5A

ii) E=IR+Ir
Ir=E-IR
Ir=12-1.5x6
Ir=3V

iii)P=IV
=1.5x9
=13.5W

b) As E=IR=Ir then if there is a greater power over the lamp then IR must have increased. As E is constant and the current is the same throughout the circuit then r must have decreased.

26.a) increases as a curve before levelling off at 12 v

b)R=V/I
= 12/2x10^-3
= 6000ohms

c)i)As V=IR and as voltage and resistance is the same so is the current.

ii) Smaller as Q=It and if the time taken is smaller to discharge then the charge is smaller. As C=Q/V as the charge is smaller and the voltage constant then the capacitance is smaller

27.a)R1/R2=R3/R4
R1/40=240/80
80R1=9600
R1=120ohms

b)i) Differential mode
ii)gain=Rf/R1
=560/100
=5.6

ii)A) Vo=(V2-V1)xgain
V2-V1=1.93V

B) V2=2.25
2.25-V1=1.93
-V1=-0.32
V1=0.32V

V1=(R1/R1+R2)xVs
4.5+0.32=(120/120+R2)x9
120/120+R2=0.535
64.267+0.535R2=120
0.535R2=55.73
R2=104.17Ohms
I took it to be 104Ohms for graph which came to be 66mm of fabric.

28.a)n=sinX/sin2
1.33=sinX/sin36
sinX=0.781...
X=51.4degrees

b)i)As there is a refracted ray at 90 degrees to the normal as well as a reflected ray

ii)sinc=1/n
sinc=1/1.33
c=48.8degrees

c) sketch of total internal reflection occuring.


29.a) nlambda=dsinA
3x633x10-9=dsin35.3
d=1.899x10-6/sin35.3
d=3.29x10-6M

b)lines per m=1/3.29x10^-6
=304295lines per M

c)102% of 3x10^5=30600
As the figure calculated in part b) is less than this figure then it satisfies the manufacturers claim.

30.a) decreases

b)i)photoconductive mode.

ii) The current increases as more photons acting on the plate and this creates more electron hole pairs available for conduction and thus the current increases.


c)Irradiance is directly proportianal to current


so I1d1^2=I2d2^2
3x10-6x1.2^2=I2x0.8^2
0.64I2=4.32x10-6
I2=6.75x10-6A

31.a) E=Dm
=500x10-6x0.04
=2x10-5J

b) H=Hdotxt
=2x5x10-3
=10x10-3sv

H=Dwr
wr=H/D
wr=10x10-3/500x10-6
=20
so radiation is alpha.


If this is all right it got 96% :biggrin:
Original post by S119234
See for 22(a)(ii) as well, is the acceleration value negative or positive ?, I used a positive value but think that might be regarded as wrong physics.

Reckon though I'll get an A if the SQA lower the pass mark ever so slightly (literally like about 3 or 4 marks!), which they might do considering the horrible multi-choice.


Yes it subtracts from ut as original velocity is upwards and acceleration naturally downwards, wether you call it negative or positive is irrelevant its the numerical answer that counts here.
paper and worked solutions:
http://www.thestudentroom.co.uk/showpost.php?p=37811443&postcount=694
(edited 11 years ago)
Original post by tomctutor
Yes it subtracts from ut as original velocity is upwards and acceleration naturally downwards, wether you call it negative or positive is irrelevant its the numerical answer that counts here.


Thank goodness!, was a bit worried then, thank you very much! :smile:
Reply 713
Original post by Direct15
I have about 71-72% after my stupid mc mistakes :frown:

Anybody know how hard it is nowadays in order to have an successful appeal?


This was posted from The Student Room's iPhone/iPad App


Very hard. They only really do it these days if there is a very serious reason. Like if you were medically unfit to sit the exam, but got a great prelim grade, for example. Even then they're still not keen to be honest. I think they're stopping appeals completely soon! This is all according to my teachers by the way.
Reply 714
Original post by S119234
Did indeed!, would love to finally get an A at Higher, here's hoping! :smile:

I'm in a similar situation to you right now, went through it and got about 72-73% on it, but considering I'm marking quite harshly, I think it might go up with odd half-marks for formula on questions I got wrong, could be the case with you, so not all hope is lost!


After maths I hold little hope, I can see the head of maths booting me out of adv maths :frown: I also got the diagram for the displacement wrong but carries through my mistakes theres hope :smile:


This was posted from The Student Room's iPhone/iPad App
Is this triple edexcel?


This was posted from The Student Room's iPhone/iPad App
Reply 716
Original post by Kaaal
Very hard. They only really do it these days if there is a very serious reason. Like if you were medically unfit to sit the exam, but got a great prelim grade, for example. Even then they're still not keen to be honest. I think they're stopping appeals completely soon! This is all according to my teachers by the way.


Would itchy sun burn count as an illness :biggrin:


This was posted from The Student Room's iPhone/iPad App
Original post by randomhero12
Are the solutions that were posted all right? Because if so I got 93% and I'm pretty chuffed with that


Myself tomctutor also Webbster91 and someone else seems to all agree, if any further errors pointed out in thread I will ammend the original docs as pointed to by worked solutions and paper scans:
http://www.thestudentroom.co.uk/showpost.php?p=37811443&postcount=694
(edited 11 years ago)
Original post by tomctutor
26.a) increases as a curve before levelling off at 12 v

b)R=V/I
= 12/2x10^-3
= 6000ohms


For question b) Is that not the total resistance so to find resistance of R you would take away the 1000 ohm resistor away from 6000 as they are in series?
(edited 11 years ago)
Original post by Direct15
After maths I hold little hope, I can see the head of maths booting me out of adv maths :frown: I also got the diagram for the displacement wrong but carries through my mistakes theres hope :smile:


EXACT same here, completely mucked up that scale drawing and got such a stupid value for displacement which I then used throughout the whole Q21. I'm also hoping that the methods will be credited, cause I would have got the right answers if the value I calculated for displacement wasn't wrong!

Quick Reply

Latest

Trending

Trending